You are on page 1of 27

Fortinberry - HNBS, UE, LE (Chapter 1 & c.

The posterior cruciate ligament is attached to the


lateral meniscus and not to the medial meniscus.
2: APK + SPT Questions)
d. The posterior cruciate ligament helps with medial
1. During the opening of a patient's mouth, a
rotation of the tibia during full knee extension with
palpable and audible click is discovered in the left
open-chain activities.
temporomandibular joint. The physician informs the
therapist that the patient has an anteriorly Ratio: The posterior cruciate ligament becomes
dislocated disk. This click most likely signifies that tight in full knee extension. This assists the tibia in
external rotation, which is needed for the screw
a. The condyle is sliding anteriorly to obtain normal
home mechanism with open-chain activities.
relationship with the disk
5. Components of lower extremity alignment that
b. The condyle is sliding posteriorly to obtain
contribute to toe in include
normal relationship with the disk
a. Femoral retroversion
c. The condyle is sliding anteriorly and losing
normal relationship with the disk b. Femoral anteversion
d. The condyle is sliding posteriorly and losing c. Calcaneovalgus feet
normal relationship with the disk d. External tibial torsion
Ratio: In the case of a reciprocal click, the initial click Ratio: Femoral anteversion is the only one that
is created by the condyle slipping back into the would account for the internal rotation seen in toe
correct position under the disk with opening of the in.
mouth. In this disorder, the condyle is resting 6. Osteochondritis dissecans occurs most commonly
posterior to the disk before jaw opening. With in the
closing, the click is caused by the condyle slipping
away from the disk. a. Capitellum

2. In what position should the therapist place the b. Humeral condyle


upper extremity to palpate the supraspinatus c. Medial femoral condyle
tendon? d. Lateral femoral condyle
a. Full abduction, full flexion, and full external Ratio: The medial femoral condyle is the most
rotation common area for osteochondritis dissecans,
b. Full abduction, full flexion, and full internal although it can occur in the femoral capital
rotation epiphysis.
c. Full adduction, full external rotation, and full 7. Considering an injury to the medial collateral
extension ligament (MCL) of the knee, when does the
d. Full adduction, full internal rotation, and full inflammatory phase of healing begin?
extension a. First days after injury
Ratio: The supraspinatus tendon is best palpated by b. 2 to 3 weeks after injury
placing the patient's involved upper extremity c. 4 to 6 weeks after injury
behind the back in full internal rotation.
d. 6 to 8 weeks after injury
3. A 13-year-old girl has fractured the left patella
Ratio: The healing phase of ligaments is divided into
during a volleyball game. The physician determines
the following categories: inflammatory phase (first
that the superior pole is the location of the fracture.
few days after injury, proliferative phase (1 to 6
Which of the following should be avoided in early
weeks after injury), and remodeling phase (begins
rehabilitation?
at 7 weeks postinjury).
a. Full knee extension
8. The cuboid bone is located just posterior to the
b. 45 degrees of knee flexion
a. Base of the first metatarsal
c. 90 degrees of knee flexion
b. Head of the first metatarsal
d. 15 degrees of knee flexion
c. Medial cuneiform bone
Ratio: The superior pole is in most contact at
d. Tuberosity of the fifth metatarsal
approximately 90 degrees of knee flexion.
Ratio: The tuberosity of the fifth metatarsal is at the
4. A 17-year-old athlete has just received a posterior
base of the metatarsal. The base of the metatarsal
cruciate ligament reconstruction. The therapist is
is proximal and the head is distal. The medial
attempting to explain some of the characteristics of
cuneiform is on the high medial side of the
the posterior cruciate ligament. Which of the
transverse arch of the foot. The cuboid bone is on
following is incorrect information?
the lower lateral side of the foot.
a. The posterior cruciate ligament prevents
9. The metacarpophalangeal joints are classified as
posterior translation of the tibia on the femur.
what type of joint?
b. Posterior bands of the posterior cruciate ligament
a. Plane
are their tightest in full knee extension.
b. Hinge
c. Condyloid Ratio: This is the normal anatomy, lateral to medial
d. Saddle of the proximal row of the carpus. The distal row,
lateral to medial, is the trapezium, trapezoid,
Ratio: Metacarpophalangeal joints are condyloid capitate, and hamate.
joints. These are biaxial joints that allow
flexion/extension around one axis and 15. A therapist is testing key muscles on a patient
abduction/adduction around another axis. who recently suffered a spinal cord injury. The
current test assesses the strength of the long toe
10. Which muscle would move the abducted (90 extensors. Which nerve segment primarily
degree) arm anteriorly? innervates this key muscle group?
a. Sternocostal head of the pectoralis major a. L2
b. Clavicular head of the pectoralis major b. L3
c. Inferior fibers of the serratus anterior c. L4
d. Pectoralis minor d. L5
Ratio: These actions can be easily demonstrated Ratio: The long toe extensors represent the spinal
and palpated. Resisting anterior movement of the cord segment L5. The iliopsoas represents L2. The
arm abducted to 60 degrees tests the sternocostal quadriceps are innervated by L3 and the tibialis
head of the pectoralis major; the clavicular head is anterior is innervated by L4.
tested after the arm is abducted to 90 degrees.
16. A physician notes a vertebral fracture in the x-
11. Which one of the following structures does NOT ray of a patient involved in a car accident. The
pass through the foramen magnum of the occipital fractured vertebra has a bifid spinous process.
bone? Which of the following vertebrae is the most likely
a. Spinal cord to be involved?
b. Meninges a. Fourth lumbar vertebra
c. Cranial nerve XII b. Fifth cervical vertebra
d. Vertebral artery c. Twelfth thoracic vertebra
Ratio: The structures that pass through the foramen d. First sacral vertebra
magnum include the spinal cord, the meninges, the Ratio: Bifid spinous processes (spinous processes
spinal components of cranial nerve XI, and the that are split) are found only in the cervical spine.
vertebral arteries. Cranial nerve XII exits the skull
through the hypoglossal canals. 17. If the line of gravity is posterior to the hip joint
in standing, on what does the body first rely to keep
12. Contraction of which muscle produces extension the trunk from moving into excessive lumbar
of the head?
extension?
a. Spinalis cervicis a. Iliopsoas muscle activity
b. Longus capitis b. Abdominal muscle activity
c. Longus colli c. Anterior pelvic ligaments and the hip joint capsule
d. Sternocleidomastoid d. Posterior pelvic ligaments and the hip joint
Ratio: The longus capitis, longus colli, and capsule
sternocleidomastoid muscles are all associated with Ratio: In static standing, the line of gravity is
the anterior aspect of the cervical vertebrae and posterior to the hip joint. The body relies on the
thus produce flexion of the head. anterior pelvic ligaments and the hip joint capsule.
13. The nucleus pulposus is thickest in which region The iliopsoas may be recruited at times, but
of the spine? anterior ligaments are used first to keep the trunk
a. Lumbar spine from extending in static stance.
b. Inferior half of the thoracic spine 18. What is the closed-packed position of the
shoulder?
c. Superior half of the thoracic spine
a. Internal rotation and abduction
d. Cervical spine
b. External rotation and abduction
Ratio: The nucleus pulposus is thickest in the lumbar
spine, followed by the cervical region; it is thinnest c. Internal rotation and adduction
in the thoracic spine. d. External rotation and adduction
14. Which of the following describes the proper Ratio: The area of contact between the humerus
normal anatomy of the proximal carpal row, from and the glenoid fossa is maximal in this position.
lateral to medial? 19. A patient with a diagnosis of a rotator cuff tear
a. Capitate, lunate, triquetrum, pisiform has just begun active range of motion. The therapist
b. Lunate, trapezium, capitate, hamate is strengthening the rotator cuff muscles to increase
joint stability and oppose the superior shear of the
c. Scaphoid, lunate, triquetrum, pisiform
deltoid. Which of the rotator cuff muscles
d. Scaphoid, hamate, lunate, capitate
participate least in opposing the superior sheer d. Foot planted, lateral tibial rotation, and no
force of the deltoid? audible pop
a. Infraspinatus Ratio: Choice B best describes the position that
b. Subscapularis causes injury only to the ACL. In most cases an
audible pop indicates a tear of the ACL. Varus or
c. Teres minor valgus blows to the knee injure the collateral
d. Supraspinatus ligaments and possibly the ACL.
Ratio: The subscapularis, teres minor, and 24. A 27-year-old woman is referred to a physical
infraspinatus muscles oppose the superior pull of therapy clinic with a diagnosis of torticollis. The
the deltoid muscle. The supraspinatus does not right sternocleidomastoid is involved. What is the
oppose the pull of the deltoid but is important most likely position of the patient's cervical spine?
because (along with the other cuff muscles) it A. Right lateral cervical flexion and left cervical
provides a compression force to the glenohumeral rotation
joint.
b. Right cervical rotation and right lateral cervical
20. What portion of the adult knee meniscus is flexion
vascularized?
c. Left cervical rotation and left lateral cervical
a. The outer edges are vascularized. flexion
b. The inner edges are vascularized. d. Left lateral cervical flexion and right cervical
c. The entire meniscus is vascular. rotation
d. The entire meniscus is avascular. Ratio: Torticollis involving the right
Ratio: Only the edges of the adult meniscus are sternocleidomastoid would cause right lateral
vascularized by the capillaries from the synovial cervical flexion and left cervical rotation.
membrane and joint capsule. 25. Observing a patient in a standing position, the
21. At what age does a human have the greatest therapist notes that an angulation deformity of the
amount of fluid in the intervertebral disc? right knee causes it to be located medially in
relation to the left hip and left foot. This condition is
a. 1 year
commonly referred to as
b. 4 years
a. Genu varum
c. 7 years
b. Genu valgum
d. 10 years
c. Pes cavus
Ratio: The intervertebral disc has the greatest
d. None of the above
amount of fluid at the time of birth. The fluid
content decreases as a person ages. Ratio: Genu valgum is a term used to describe a
deformity of the knee causing an inward bowing of
22. Which of the following is not an example of a
the legs. Genu varus is an outward bowing of the
synarthrodial joint in the body?
legs. Coxa valgum is a deformity at the hip in which
a. Coronal suture the angle between the axis of the neck of the femur
b. The fibrous joint between the shaft of the tibia and the shaft of the femur is greater than 135
and fibula degrees. In coxa varus this angle is less than 135
c. Symphysis pubis degrees. Pes cavus is an increase in the arch of the
foot. Pes planus is flat foot.
d. Metacarpophalangeal
26. Which of the following is the most vulnerable
Ratio: The metacarpophalangeal joint is enclosed in position for dislocation of the hip?
a joint capsule and therefore is considered a
diarthrodial joint. a. 30 degrees hip extension, 30 degrees hip
adduction, and minimal internal rotation
23. A football player presents to an outpatient clinic
with complaints of pain in the right knee after an b. 30 degrees hip flexion, 30 degrees hip adduction,
injury suffered the night before. The physician and minimal external rotation
determines that the anterior cruciate ligament (ACL) c. 30 degrees hip flexion, 30 degrees hip abduction,
is torn. Which of the following is most commonly and minimal external rotation
associated with an injury causing damage to the ACL d. 30 degrees hip extension, 30 degrees hip
only? abduction, and minimal external rotation
a. Varus blow to the knee with the foot planted and Ratio: This is the loose-packed position of the hip.
an audible pop
27. Which of the following articulate with the
b. Foot planted, medial tibial rotation, and an second cuneiform?
audible pop
a. Navicular
c. Valgus blow to the knee with the foot planted and
b. Talus
no audible pop
c. First metatarsal
d. Cuboid
Ratio: The second cuneiform of the foot articulates following observations is not true in examining a
with the first cuneiform, second metatarsal, third patient without foot or ankle problems in the
cuneiform, and navicular. standing position?
28. A physician ordered a splint for a patient who a. The talus is situated somewhat medially to the
should keep the thumb of the involved hand in midline of the foot.
abduction. A new graduate is treating the patient b. In quiet standing, the muscles surrounding the
and is confused about the difference between ankle joint remain silent.
thumb flexion, extension, abduction, and adduction.
Which of the following lists is correct? c. The first and second metatarsal heads bear more
weight than the fourth and fifth metatarsal heads.
a. Extension is performed in a plane parallel to the
palm of the hand, and abduction is performed in a d. The talus transmits weight to the rest of the
bones of the foot.
plane perpendicular to the palm of the hand.
b. Flexion is performed in a plane perpendicular to Ratio: Plantar flexors have to contract in quiet
standing. Other muscles are recruited with
the palm of the hand, and adduction is performed in
a plane parallel to the palm of the hand. movement of the center of gravity.
32. A physical therapist is examining a female
c. Extension is performed in a plane perpendicular
to the palm of the hand, and adduction is distance runner who complains of intermittent
medial ankle pain. In static standing, the therapist
performed in a plane parallel to the palm of the
hand. palpates excessive lateral deviation of the head of
the talus. From this information, in what position is
d. In referring to positions of the thumb, flexion and the subtalar joint during palpation?
adduction are used synonymously, and extension
a. Supination
and abduction are used synonymously.
Ratio: Flexion and extension of the thumb are b. Pronation
performed in a plane parallel to the palm of the c. Neutral
hand. Abduction and adduction are performed in a d. Unable to determine from the information given
plane perpendicular to the palm of the hand.
Ratio: The talus is palpated just anterior and lateral
29. A therapist is assisting a patient in gaining lateral to the medial malleolus. Supination is excessive
stability of the knee joint. The therapist is using lateral deviation of the talus, and pronation is
strengthening exercises to strengthen muscle excessive medial deviation.
groups that will increase active restraint on the
33. Which of the following is not part of the
lateral side of the joint. Which of the following
triangular fibrocartilage complex of the wrist?
offers the least amount of active lateral restraint?
a. Dorsal radioulnar ligament
a. Gastrocnemius
b. Ulnar collateral ligament
b. Popliteus
c. Radial collateral ligament
c. Biceps femoris
d. Ulnar articular cartilage
d. Iliotibial band
Ratio: The triangular fibrocartilage complex is made
Ratio: The popliteus, biceps femoris, and iliotibial
up of the dorsal radioulnar ligament, ulnar collateral
band offer active restraint for the lateral side of the
ligament, ulnar articular cartilage, volar radioulnar
knee joint. The gastrocnemius assists in active
ligament, ulnocarpal meniscus, and sheath of the
restraint of the posterior side of the knee joint.
extensor carpi ulnaris.
30. A patient is in an outpatient facility because of
34. Temporomandibular anterior disc displacement
an injury sustained to the right knee joint. Only the
without reduction occurs between the
structures within the synovial cavity were
compromised during the injury. Knowing this a. Disc and the lower joint compartment
information only, the therapist is not concerned b. Disc and the ementia articularis
with injury to which of the following structures? c. Disc and the lateral pterygoid muscle
a. Patellofemoral joint d. Disc and the upper joint compartment
b. Anterior cruciate ligament Ratio: The condyle glides anterior translatory down
c. Medial meniscus the eminence (26 mm to 50 mm of opening) with
d. Femoral condyles the disc in the upper joint compartment.
Occasionally the disc is displaced anteriorly, and
Ratio: The anterior cruciate ligament is located
adhesions may occur that produce an anterior
within the articular cavity but outside the synovial
displaced disc without reduction (locked joint).
lining. The anterior and posterior cruciate ligaments
have their own synovial lining. 35. What are the signs and symptoms of a
temporomandibular anterior displaced disc with
31. A patient is being examined by a physical
reduction?
therapist because of bilateral knee pain. The
therapist is attempting to rule out ankle or foot a. Crepitation with loss of opening
dysfunction as the source of the pain. Which of the b. Clicking with opening
c. No clicking with loss of opening Ratio: Shortened hip flexors and lengthened
d. Temporomandibular joint tenderness and loss of anterior abdominals will contribute to an anterior
opening tilt. Choices A and D would create a posterior pelvic
tilt, and choice B will likely present with a more
Ratio: Reduction indicates that the condyle is able neutral position of the pelvis.
to slide under the disc (reduce), causing a clicking
noise. 41. A patient presents with anterior knee pain,
which of the following cannot be the source of that
36. What is the normal TMJ arthrokinematics for pain?
lateral movements?
a. Synovium
a. Bilateral translation
b. Capsule
b. Bilateral rotation
c. Patella cartilage
c. Contralateral rotation and ipsilateral translation
d. Patella bone
d. Ipsilateral rotation and contralateral translation
Ratio: Patella cartilage cannot be the source of the
Ratio: Ipsilateral rotation and contralateral pain. Every structure in the knee has pain nerve
translation describe lateral jaw movement. fibers except the articular cartilage.
37. What is the normal TMJ arthrokinematics for 42. Anterior cruciate ligament tears do not heal as
protrusion? well as medial collateral ligament tears because
a. Bilateral anterior translation a. The ACL is under greater tension
b. Bilateral posterior translation b. Synovial fluid inhibits ACL healing
c. Ipsilateral rotation with contralateral translation c. The MCL is broad and flat allowing better healing
d. Bilateral rotation d. There is more motion in the ACL
Ratio: Bilateral anterior translation describes jaw Ratio: Synovial fluid has been shown to inhibit
protrusion. healing of ligament tissues.
38. What is the normal TMJ arthrokinematics for 43. Patellofemoral joint reactive forces are highest
wide opening? with
a. Bilateral translation a. Running
b. Combination of rotation occurs first 26 mm then b. Straight leg raises
anterior translation
c. Prolonged sitting
c. Combination of anterior translation occurs first 26
mm then anterior rotation d. Plyometrics
d. Bilateral rotation Ratio: Jumping (plyometrics) can generate up to 7
times body weight at the patella. Running is next
Ratio: A combination of rotation occurs during the most stressful at 3.5 times body weight.
first 26 mm, then anterior translation describes
wide jaw opening. 44. A patient was referred for physical therapy after
removal of a long arm cast extending to the
39. Positioning of a patient in right side-lying can forearm. She lacks full passive elbow extension.
create pressure on the What may be causing this problem?
a. Right ischial tuberosity a. Active insufficiency of the biceps
b. Left greater trochanter b. Tightness in posterior humeroulnar joint capsule
c. Right lateral malleolus c. Passive insufficiency of the pronator teres
d. Occiput d. Passive insufficiency of the triceps
Ratio: Pressure in right side-lying would create Ratio: Passive insufficiency of the pronator teres.
pressure on the right greater trochanter and right The question asks specifically about passive
lateral mallelous. No pressure is put on the occiput limitations to elbow extension. The pronator teres is
or ischial tuberosity in side-lying. a polyarticular muscle crossing anterior to the
40. Ideal postural alignment is influenced by elbow joint.
appropriate muscle balance. What combinations of 45. A 10 degrees hip flexion contracture produces
muscle imbalance would likely contribute to _____ torque at the hip that increases muscle
increased anterior pelvic tilt? demand on the _____.
a. Short hamstrings and elongated hip flexors a. Extension, quadriceps
b. Strong anterior abdominals and strong hip flexors b. Flexion, biceps femoris
c. Short hip flexors and lengthened anterior c. Abduction, adductor magnus
abdominals
d. Flexion, iliopsoas
d. Strong anterior abdominals and strong hip
extensors Ratio: Choice B, flexion, biceps femoris, is the
correct answer. A hip flexion contracture increases
flexor torque across the anterior hip. This increases
muscle demand on the hip extensors. The biceps a. Sural nerve
femoris is an extensor of the hip. Passive b. Deep fibular
insufficiency of the pronator teres. The question
c. Tibial nerve
asks specifically about passive limitations to elbow
extension. The pronator teres is a polyarticular d. Saphenous nerve
muscle crossing anterior to the elbow joint. Ratio: The saphenous nerve lies just anterior to the
46. The physical therapist is analyzing a patient's medial malleolus alongside the great saphenous
gait with descending stairs. During left single limb vein. This nerve is most likely to be injured by the
stance, the patient demonstrates a right pelvic drop sutures.
with left trunk lean. The physical therapy hypothesis 50. The neural canal is smallest and circular in shape
is in the _____ region of the vertebral canal.
a. Weak right gluteus medius with left trunk lean to a. Cervical
move center of mass towards stronger side
b. Thoracic
b. Weak left gluteus medius with left trunk lean to
c. Lumbar
move center of mass towards weaker side
d. Sacral
c. Weak left quadratus lumborum producing left
trunk lean Ratio: The neural canal (vertebral canal) is largest
and most triangular in the cervical region and
d. Weak right gluteus medius with left trunk lean to
smallest and most circular in the thoracic region.
move center of mass towards stronger side
51. A posterior lateral herniation of the lumbar disc
Ratio: Weak left gluteus medius with left trunk lean
between vertebrae L4 and L5 most likely results in
to move center of mass towards weaker side is the
damage to which nerve root?
correct hypothesis. A pelvic drop in single limb
midstance is a classic positive Trendelenberg sign of a. L4
gluteus medius weakness. The compensatory trunk b. L5
lateral lean is to bring the center of mass closer to c. L4 and L5
the weaker side to decrease the external moment
arm on the weak muscle. d. L5 and S1
47. Left lateral trunk flexion is limited primarily by Ratio: The fifth lumbar nerve root is impinged
because it arises from the spinal column superior to
a. The thoracic spine because of sagittal facet the L4-L5 lumbar disc.
alignment
52. A patient has traumatically dislocated the tibia
b. The rib cage because of multiple attachments directly posteriorly during an automobile accident.
c. The lumbar spine because of horizontal plane Which of the following structures is the least likely
facet orientation to be injured?
d. The left quadratus lumborum a. Tibial nerve
Ratio: Left lateral trunk flexion is limited primarily b. Popliteal artery
by the rib cage because of multiple attachments. c. Common peroneal nerve
The thoracic spine has frontal plane facet
alignment. The lumbar spine facets are oriented in d. Anterior cruciate ligament
the sagittal plane. The left quadratus lumborum is Ratio: The common peroneal nerve travels over the
on slack during left lateral flexion. lateral knee. It is the least likely to be injured. The
48. Which is true about the hip joint? other structures are either within the knee or
directly posterior.
a. The hip joint's closed pack position is extension
with full external rotation. 53. A patient is referred to physical therapy with
complaints of sensation loss over the area of the
b. The hip joint's loose pack position is 30 degrees radius of the right upper extremity, extending from
of abduction, 70 degrees of flexion with lateral the elbow joint distally to the wrist. Therapy
rotation. sessions are focused on assisting the patient in
c. With its capsular pattern of restriction, medial regaining normal sensation. Which of the following
rotation is most restricted in the hip joint. nerves is responsible for sensation in this region?
d. With its capsular pattern of restriction, flexion is a. Medial antebrachial cutaneous
most restricted in the hip joint. b. Lateral antebrachial cutaneous
Ratio: According to Cyriax's classical description, c. Musculocutaneous
flexion in the hip is limited to the greatest in its
capsular pattern. d. Both B and C
49. Your patient had a laceration anterior to the Ratio: The radial side is the lateral side of the
medial malleolus that required stitches. He is now in forearm, which is innervated by the
your office complaining of pain along the medial musculocutaneous nerve. The lateral antebrachial
border of the foot. Which nerve is most likely cutaneous nerve is a continuation of the
involved? musculocutaneous nerve.
54. A physical therapist working in an early Ratio: Increased external rotation range of motion.
intervention program is providing intervention to an Excessive femoral anteversion has been shown to
infant diagnosed with Erb's palsy. This condition be related to increased internal rotation and
most often involves what nerve roots? decreased external rotation ROM. Also, it has been
a. C2-C3 shown that it results in a reduced hip abductor
moment arm.
b. C3-C4
59. Your patient has a lesion of the left superior
c. C5-C6 gluteal nerve. When your patient is in left unilateral
d. C8-T1 stance you may observe the
Ratio: The most common injury of the brachial a. Right ASIS is higher than the left ASIS
plexus is to the upper roots, C5-C6, resulting in an b. Right ASIS is anterior to the left ASIS
Erb's palsy.
c. Trunk side bending to the left
55. A patient with an Erb's palsy will have paralysis
of all of the following muscles except the d. Lumbar spine is side bent to the right
a. Flexor carpi ulnaris Ratio: Trunk side bending to the left. The superior
gluteal nerve innervates the gluteus medius muscle.
b. Rhomboids A lesion would result in a Trendelenburg stance,
c. Brachialis leading to left side bending of the spine (to maintain
d. Teres minor an upright posture).
Ratio: An injury to the C5-C6 nerve roots results in 60. Standing on the left leg and flexing your right hip
an Erb's palsy. The flexor carpi ulnaris is innervated up requires you to use all the following muscles
by C8-T1. except the
56. Whiplash injury from a rear-end collision would a. Right lumbar rotators
tear the b. Left gluteus minimus
a. Posterior longitudinal ligament (PLL) c. Right quadratus lumborum
b. Anterior longitudinal ligament (ALL) d. Left gluteus medius
c. Ligamentum nuchae Ratio: Choice A, right lumbar rotators, is the correct
d. Ligamentum flavum answer. The single leg stance would result in
activation of all of the left-sided hip abductors and
Ratio: Whiplash injury includes hyperextension of
the right quadratus lumborum.
cervical vertebrae that may tear the anterior
longitudinal ligament that limits extension of the 61. A smaller than normal angle of inclination at the
cervical spine. All of the other ligaments limit flexion hip is called
of the cervical spine; accordingly, they may be torn a. Anteversion
in hyperflexion injuries. b. Retroversion
57. A physical therapist should place the knee in c. Coxa vara
which of the following positions to palpate the
lateral collateral ligament (LCL)? d. Coxa valga
a. Knee at 60 degrees of flexion and the hip Ratio: Choice C, coxa vara, is the correct answer.
externally rotated Coxa valga would be a larger than normal angle of
inclination at the hip. Anteversion is the angle made
b. Knee at 20 degrees of flexion and the hip at by the femoral neck and the femoral condyles (as
neutral measured from the coronal plane). Excessive medial
c. Knee at 90 degrees of flexion and the hip rotation is anteversion, and excessive lateral
externally rotated rotation is retroversion.
d. Knee at 0 degrees and the hip at neutral 62. Hip extension may be limited by all of the
Ratio: The lateral collateral ligament of the knee is following tissues except
best palpated with the patient in the sitting a. Iliofemoral ligament
position. The patient then places the foot of the b. Iliopsoas muscle
involved lower extremity on the knee of the
uninvolved lower extremity. This maneuver places c. Ischiofemoral ligament
the involved knee in 90 degrees of flexion and the d. Gluteus minimus posterior fibers
hip in external rotation. Ratio: Choice D, gluteus minimus posterior fibers, is
58. Excessive femoral anteversion in children may the correct answer. Hip abductors and hip extensors
result in all of the following except do not limit hip extension. All the other choices may
a. Toeing in during gait limit hip extension.
b. Increased hip internal rotation range of motion 63. An angle of 170 degrees to 175 degrees in the
frontal plane taken on the lateral side of the knee is
c. Increased external rotation range of motion considered
d. Decreased external rotation range of motion a. Excess genu valgum
b. Excess genu varum relationship could signal deficiencies in the rotator
c. Normal cuff musculature.
d. Coxa vara 68. Anterior glenohumeral dislocations are often
accompanied by
Ratio: Choice C, normal, is the correct answer. Genu
valgum is an abnormal inward "bowing" of the a. A stretched subscapularis
knees (knock knee). Genu varus is an outward b. A Hill-Sachs lesion
anatomic presence of the knee (bow-legged). c. A fracture of the greater tubercle
64. The pes anserine insertion is palpable at the d. Rotator cuff full-thickness tears
________ and includes the tendons of the ______.
Ratio: Choice B, a Hill-Sachs lesion, is the correct
a. Medial tibia, semimembranosus, semitendinosis, answer. While many of these injuries are seen with
and gracilis sholder dislocation, a Hill-Sachs lesion is by far the
b. Lateral tibia, biceps femoris, semitendinosus, and most common. A Hill-Sachs lesion to the humerus is
iliotibial band caused when the smooth surface of the humerus
c. Medial tibia, sartorius, gracilis, and hits the outer rim of the glenoid fossa.
semitendinosus 69. Which of the following is false regarding
d. Medial femur, biceps femoris, semitendinosus, biomechanics of persons with patellofemoral pain?
and iliotibial band a. Weakness of the hip abductors, external rotators,
Ratio: Choice C, medial tibia, sartorius, gracilis, and and extensors is frequently present
semitendinosus, is the correct answer. An b. Excessive hip internal rotation and/or hip
interesting pneumonic to remember the pes adduction is frequently present
anserine insertion is "Say Grace before SupperTime" c. Patella alta increases patellar instability
Say = sartorius, Grace = gracilis, SupperTime =
semitendinosus. d. Increased trochlear groove depth increases
patellar instability
65. As the knee extends and the patella moves
superiorly in the trochlear groove, the sulcus angle Ratio: Increased trochlear groove depth increases
patellar instability. An internally rotated femur
_____ making the patellofemoral joint _____ stable.
causes the patella to track laterally. A lateral patella
a. Increases, less is unstable. Weakness of the hip external rotators
b. Decreases, less will lead to a more internally rotated femur. A
c. Increases, more superior patella (as in patella alta), will move the
patella out of the trochlear groove. However,
d. Decreases, more increased trochlear groove depth makes the patella
Ratio: Choice A, increases, less, is the correct MORE stable.
answer. As the patella rides superiorly out of the 70. Morton's neuroma is usually located between
trochlear groove, the joint becomes less stable which metatarsal heads?
because less of the patella is in contact with the
trochlear groove. a. First and second
66. The glenoid faces somewhat b. Second and third
a. Lateral and inferior and posteriorly c. Third and fourth
b. Lateral and superior and anteriorly d. Fourth and fifth
c. Medial and superior and anteriorly Ratio: A painful neuroma in the space between the
third and the fourth metatarsal heads is a Morton's
d. Medial and inferior and posteriorly neuroma.
Ratio: Choice B, lateral and superior and anteriorly, 71. What shoulder view BEST demonstrates the
is the correct answer. This is the normal anatomic greater tubercle?
presentation of the glenoid.
a. External rotation
67. During normal scapulohumeral rhythm, the
b. Internal rotation
a. Scapula upwardly rotates 60 degrees and the
humeral abducts 120 degrees c. Baby arm
b. Scapula upwardly rotates 2 degrees for every 1 d. Transthoracic
degree of humeral abduction Ratio: Due to the anatomic location of the greater
c. Scapula abducts 60 degrees and upwardly rotates tubercle, external rotation positions it in profile for
120 degrees best visualization.
d. Scapula upwardly rotates 120 degrees and the 72. While assessing the standing posture of a
humeral abducts 60 degrees patient, the therapist notes that a spinous process
in the thoracic region is shifted laterally. The
Ratio: The scapula upwardly rotates 60 degrees and therapist estimates that T2 is the involved vertebra
the humeral abducts 120 degrees. The scapula because he or she notes that it is at the
upwardly rotates 1 degree for every 2 degrees of approximate level of the
humeral abduction. Abnormalities of this
a. Inferior angle of the scapula most likely muscle that the therapist needs to focus
b. Superior angle of the scapula on during the exercise session in order to minimize
this gait deviation?
c. Spine of the scapula
a. Gluteus medius
d. Xiphoid process of the sternum
b. Gluteus maximus
Ratio: The superior angle of the scapula commonly
rests at the same level as vertebra T2. The spine of c. Quadriceps
the scapula is approximately at T3. The inferior d. Hamstrings
angle of the scapula and xiphoid process represent Ratio: This gait deviation is caused by the patient
T7. leaning back to decrease the flexion moment
73. Which of the following actions places the created at the hip at initial contact. The gluteus
greatest stress on the patellofemoral joint? maximus is most responsible for counteracting this
a. When the foot first contacts the ground during flexion moment.
the gait cycle 78. The therapist is performing an orthopedic test
b. Exercising on a stair-stepper machine on a 25-year-old man with the chief complaint of
low back pain. The patient has a positive Thomas
c. Running down a smooth decline of 30 degrees test. With this information, what might the
d. Squats to 120 degrees of knee flexion therapist need to include in the treatment plan?
Ratio: Patellofemoral joint reaction forces increase a. Stretching of the hip abductors
as the angle of knee flexion and quadriceps muscle b. Stretching of the hip adductors
activity increase. Choice D involves the greatest
knee flexion angle and quadriceps activity. c. Stretching of the hip extensors
74. Which of the following does the therapist d. Stretching of the hip flexors
observe if a patient is correctly performing an Ratio: The Thomas test is a screen to determine
anterior pelvic tilt in standing position? whether the hip flexors are too tight.
a. Hip extension and lumbar flexion 79. A patient is positioned by the therapist with the
b. Hip flexion and lumbar extension cervical spine rotated to the right. The patient then
extends the neck as the therapist externally rotates
c. Hip flexion and lumbar flexion and extends the right upper extremity. The patient
d. Hip extension and lumbar extension is then instructed to hold a deep breath. The radial
Ratio: Choice B is the correct answer. Choice A is a pulse is palpated in the right upper extremity by the
posterior pelvic tilt. therapist. What type of special test is this, and for
what condition is it testing?
75. A 14-year-old girl with right thoracic scoliosis is
referred to physical therapy. The therapist should a. Adson's maneuver, cervical disc herniation
expect which of the following findings? b. Lhermitte's sign, cervical disc herniation
a. Left shoulder high, left scapula prominent, and c. Adson's maneuver, thoracic outlet syndrome
right hip high d. Lhermitte's sign, thoracic outlet syndrome
b. Left shoulder low, right scapula prominent, and Ratio: Adson's maneuver tests for thoracic outlet
left hip high syndrome. Lhermitte's sign tests for dural irritation
c. Right shoulder high, right scapula prominent, and in the cervical spine.
right hip high 80. A therapist is examining the gait pattern of a
d. Right shoulder low, right scapula prominent, and patient and notes that the pelvis drops inferiorly on
left hip high the right during the midswing phase of the right
Ratio: The patient probably has a low left shoulder, lower extremity. The patient also leans laterally to
prominent right scapula, and high left hip. the left with the upper trunk during this phase.
Which of the following is the most likely cause of
76. What is the most likely cause of anterior pelvic
this deviation?
tilt during initial contact (heel strike)?
a. Weak right gluteus medius
a. Weak abdominals
b. Weak right adductor longus
b. Tight hamstrings
c. Weak left gluteus medius
c. Weak abductors
d. Weak left adductor longus
d. Back pain
Ratio: The pelvis is dropping on the right side
Ratio: Abdominal muscles attach to the lower
because the left gluteus medius is weak. The patient
border of the ribs and the superior surface of the
also may lean toward the left hip joint to move the
pelvis. Strong abdominals prevent excessive
center of gravity, making it easier to hold up the
anterior rotation of the pelvis during gait.
right side of the pelvis.
77. During examination of a patient, the therapist
81. The therapist is performing an orthopedic test
observes significant posterior trunk lean at initial
that involves (1) placing the patient in a side-lying
contact (heel strike). Which of the following is the
position, (2) placing the superior lower extremity in
hip extension and hip abduction, (3) placing the being restricted would be the flexor digitorum
knee of the superior lower extremity in 90 degrees superficialis.
of flexion, and (4) allowing the superior lower b. The fourth finger would flex at the proximal
extremity to drop into adduction. Failure of the interphalangeal (PIP) joint only, and the muscle
superior lower extremity to drop indicates a tight being restricted would be the flexor digitorum
a. Iliopsoas profundus.
b. Rectus femoris c. The fourth finger would flex at the DIP joint only,
c. Iliotibial band and the muscles being restricted would be the
lumbricals.
d. Hamstring
d. The fourth finger would flex at the PIP joint only,
Ratio: This is called Ober's test, which screens for a and the muscles being restricted would be the
tight iliotibial band. palmar interosseous.
82. A therapist is assessing radial deviation range of Ratio: The flexor digitorum profundus has four
motion at the wrist. The correct position of the tendons, each attaching to the distal phalanx. If the
goniometer should be as follows: the proximal arm three mentioned in the question are restricted,
is aligned with the forearm and the distal arm is flexion at the distal interphalangeal joint in the
aligned with the third metacarpal. What should be normal hand would not be possible.
used as the axis point?
85. A patient is in prone position with his head
a. Lunate rotated to the left side. The left upper extremity is
b. Scaphoid placed at his side and fully internally rotated. The
c. Capitate left shoulder is then shrugged toward the chin. The
therapist then grasps the midshaft of the patient's
d. Triquetrum
left forearm. The patient is then instructed to "try to
Ratio: The capitate is the axis. reach your feet using just your left arm." This
83. A therapist is screening a patient complaining of movement is resisted by the therapist. This test is
pain at the anterior left shoulder region. The pain is assessing the strength of what muscle?
increased when the examiner instructs the patient a. Upper trapezius
to position the left arm by his side with the elbow
b. Posterior deltoid
flexed at 90 degrees and to actively supinate the
forearm against resistance (provided by the c. Latissimus dorsi
examiner). What test is being performed? d. Triceps brachii
a. Froment's sign Ratio: This test assesses the strength of the
b. Yergason's test latissimus dorsi. One of the functions of the
latissimus is to push up from a sitting position. This
c. Waldron test
test simulates that movement.
d. Wilson test
86. Which of the following is the normal end-feel
Ratio: Yergason's test detects tendinitis of the long perceived by an examiner assessing wrist flexion?
head of the biceps. Froment's sign is a test to
a. Bone to bone
determine adductor pollicis weakness due to ulnar
nerve dysfunction. In the Waldron test, the patient b. Soft tissue approximation
performs squats while the therapist assesses the c. Tissue stretch
patella region for crepitus or pain. A positive test d. Empty
indicates possible chondromalacia. A positive
Wilson test indicates possible osteochondritis Ratio: A tissue stretch end-feel is also felt with ankle
dissecans. The test is performed by asking the dorsiflexion. An example of a bone-to-bone end-feel
patient to extend the knee in the seated position is with knee or elbow extension. Knee flexion is an
with internal rotation and again with external example of soft tissue approximation. In an empty
rotation of the tibia. The test is positive if there is end-feel, a patient stops the movement due to pain.
pain with internal rotation and no pain with 87. A physical therapist is beginning an examination
external rotation of the tibia. of a patient with a diagnosis of "knee strain." Range
84. A therapist is examining a patient with traumatic of motion limitation does not follow the normal
injury to the left hand. The therapist asks the capsular pattern of the knee. Which of the following
patient to place the left hand on the examination are possible causes of the restriction in range of
table with the palm facing upward. The therapist motion?
then holds the second, third, and fifth digits in full a. Ligamentous adhesions
extension. The patient is then asked to flex the b. Internal derangement
fourth digit. What movement would be expected by
a patient with an uninjured hand, and what muscle c. Extra-articular lesions
or muscles is the therapist restricting? d. All of the above
a. The fourth finger would flex at the distal Ratio: All of the choices are capable of causing a
interphalangeal (DIP) joint only, and the muscle noncapsular pattern.
88. Which of the following statements best posterior force is applied to the tibia to assess
describes lower extremity positioning in standing posterior cruciate ligament integrity. When
during the first 2 years of life of a child with no performing these tests, the therapist is assessing
dysfunction? the end-feel and amount of joint play to determine
a. Femoral anteversion, femoral external rotation, the integrity of the ligament.
foot pronation 91. A therapist is examining a patient who
b. Femoral anteversion, femoral internal rotation, complains of frequent foot, ankle, and knee pain.
foot supination The therapist asks the patient to assume a standing
position with the knees slightly flexed. The therapist
c. Femoral retroversion, femoral external rotation, then demonstrates active bilateral foot pronation to
foot pronation the patient. When asked to perform this task, the
d. Femoral retroversion, femoral internal rotation, patient has difficulty. Which of the following
foot supination limitations is a possible cause of the patient's
Ratio: After the first 2 years of life, the femurs difficulty in performing this task?
rotate to a more neutral position, and the amount a. Restriction limiting plantar flexion and lateral
of anteversion decreases. rotation of the talus
89. A patient with decreased function of the gluteus b. Restriction limiting dorsiflexion and medial
minimus is referred to physical therapy for gait rotation of the talus
training. During the examination, the therapist c. Restriction limiting eversion of the calcaneus and
places the patient in prone position and instructs medial rotation of the talus
the patient to extend the hip. Knowing that the
gluteus minimus is extremely weak, which of the d. Restriction limiting inversion of the calcaneus and
following is most likely to happen? lateral rotation of the talus
a. The patient will abduct the hip more than usual Ratio: During pronation of the feet, the calcaneus
when attempting to perform hip extension. everts, and the talus medially rotates and plantar
flexes.
b. The patient will externally rotate the hip
excessively when attempting to perform hip 92. A patient presents to therapy with an ankle
extension. injury. The therapist has determined that the injury
is at the junction of the distal tibia and fibula. Which
c. The patient will excessively flex the knee when of the following functions the most in preventing
attempting to perform hip extension. excessive external rotation and posterior
d. The patient will not have difficulty performing displacement of the fibula?
straight hip extension. a. Anterior inferior tibiofibular ligament
Ratio: The patient is prone to excessive external b. Posterior inferior tibiofibular ligament
rotation when attempting to extend the involved
hip because the gluteus minimus counteracts the c. Interosseous membrane
lateral rotational force created by the gluteus d. Long plantar ligament
maximus. Ratio: This is the primary function of the anterior
90. A patient is placed in supine position with the inferior tibiofibular ligament.
knee in 90 degrees of flexion. The foot is stabilized 93. A physical therapist is examining a patient who
by the therapist's body on the examination table. complains of posterior ankle pain. The patient is
The therapist then wraps his fingers around the positioned prone with the feet extended over the
proximal tibia so that the thumbs are resting along edge of the mat. The therapist squeezes the
the anteromedial and the anterolateral margins. involved gastrocnemius over the middle third of the
The therapist then applies a force to pull the tibia muscle belly. What test is the therapist performing?
forward. What special test is being performed? What indicates a positive test?
a. Pivot shift a. Thompson's test, plantar flexion of the ankle
b. Lachman's test b. Homan's test, plantar flexion of the ankle
c. Anterior drawer c. Thompson's test, no ankle movement
d. Posterior drawer d. Homan's test, no ankle movement
Ratio: The tests in Choices A, B, and C assess the Ratio: Thompson's test checks the integrity of the
integrity of the anterior cruciate ligament. The pivot Achilles' tendon. When this test is performed on an
shift test is performed with the patient in supine ankle with no dysfunction, squeezing the
position. The therapist applies a valgus stress with gastrocnemius causes passive plantar flexion of the
the lower leg internally rotated while passively ankle.
flexing and extending the knee. A positive test is
associated with instability with this motion. 94. Which of the following is the correct method to
test for interossei muscular tightness of the hand?
Lachman's test is similar to the anterior drawer test,
but the knee is in slight flexion. In performing a a. Passively flex the proximal interphalangeal (PIP)
posterior drawer test, the positioning is the same as joints with the metacarpophalangeal (MP) joints in
for performing an anterior drawer test, but a extension, then passively flex the PIP joints with the
MP joints in flexion. Record the difference in PIP a. The subtalar joint should be placed in pronation.
joint passive flexion. b. The subtalar joint should be placed in supination.
b. Passively extend the PIP joints with the MP joints c. The subtalar joint should be placed in a neutral
in extension, then passively extend the PIP joints position.
with the MP joints in flexion. Record the difference
d. The position of the subtalar joint does not
in PIP joint passive flexion.
influence forefoot compensation.
c. Passively flex the PIP joints with the MP joints in
Ratio: When the hindfoot is pronated, the forefoot
extension, then passively extend the PIP joints with
the MP joints in flexion. Record the difference in PIP (transverse tarsal joints) can compensate for
uneven terrain. If the hindfoot is supinated, the
joint passive flexion.
forefoot also is likely to supinate and possibly cause
d. Passively extend the PIP joints with the MP joints damage to the lateral ankle ligaments.
in extension, then passively flex the PIP joints with
the MP joints in flexion. Record the difference in PIP 98. In the terminal swing phase of gait, what
muscles of the foot and ankle are active?
joint passive flexion.
Ratio: Because the interossei cross the MP joints a. Extensor digitorum longus
and the PIP joints, the PIP joints should be flexed b. Gastrocnemius
with the MP joints in flexion and extension. c. Tibialis posterior
95. A therapist is beginning an examination of a 34- d. Flexor hallucis longus
year-old woman with a diagnosis of carpal tunnel
Ratio: The tibialis anterior, extensor digitorum
syndrome. Part of the evaluation consists of grip
longus, and extensor hallucis longus contract
strength testing. To accurately test strength of the
concentrically to achieve a neutral ankle position
flexor digitorum profundus, where should the grip
before initial contact.
dynamometer's adjustable handle be placed?
99. When the knee is at its maximal amount of
a. 1 inch from the dynamometer's nonadjustable
flexion during the gait cycle, which of the following
handle
muscles are active concentrically?
b. 3 inches from the dynamometer's nonadjustable
a. Hamstrings
handle
b. Gluteus maximus
c. 1.5 inches from the dynamometer's
nonadjustable handle c. Gastrocnemius
d. 4 inches from the dynamometer's nonadjustable d. Flexor hallucis longus
handle Ratio: The hamstrings bring the knee to
Ratio: The last position (3 inches) of the grip approximately 60 degrees of flexion during
strength dynamometer tests the extrinsic muscles acceleration. The hip flexors, ankle dorsiflexors, and
of the hand (muscles located in the forearm). The toe extensors are also active.
closer positions test the intrinsic muscles. 100. The alar ligament stress test is considered
96. A physical therapist is examining a 17-year-old positive if
distance runner with complaints of lateral knee a. Laxity is felt in neutral
pain. During the evaluation, the therapist performs b. Laxity is felt in extension
the following test: The patient is placed in supine
position with the hip flexed to 45 degrees and the c. Laxity is felt in flexion
knee to 90 degrees. The therapist then places firm d. Laxity is felt in both flexion and extension
pressure over the lateral femoral epicondyle and Ratio: The test is only considered positive if it is lax
extends the patient's knee. Pain is felt by the in two or more planes. This is due to the variation of
patient at the point of palpation when her knee is direction of the fibers as they connect to the alar
30 degrees from full knee extension. The positive ligament.
result of this test suggests which of the following
101. A physical therapist is examining a patient
structures as the source of pain?
reporting knee pain. The patient is positioned in a
a. Iliotibial band prone position, and the physical therapist passively
b. Biceps femoris flexes the knee to end range. Based on the
c. Quadriceps examination technique, which of the following
structures would not be expected to limit
d. Lateral collateral ligament movement?
Ratio: The patient has an irritation of the iliotibial a. Joint capsule
band as it passes over the lateral femoral
epicondyle. This occurs at approximately 30 degrees b. Vastus intermedius
from full knee extension. c. Sciatic nerve
97. When ambulating on uneven terrain, how d. Rectus femoris
should the subtalar joint be positioned to allow Ratio: Passive flexion of the knee would be
forefoot rotational compensation? expected to place stress on the muscles acting to
extend the knee and the joint capsule. The only a painful arc between 70 degrees and 120 degrees
tissue structure that would be placed on "slack" of active abduction in the involved shoulder. This
would be the sciatic nerve, which runs along the finding is most indicative of what shoulder
posterior aspect of the femur. pathology?
102. The L4 deep tendon reflex is elicited at the a. Rotator cuff tear
a. Achilles tendon b. Acromioclavicular joint separation
b. Femoral tendon c. Impingement
c. Medial hamstring tendon d. Labrum tear
d. Patella tendon Ratio: The "painful arc" is most indicative of
Ratio: According to Hoppenfeld, the patella deep shoulder impingement. The soft tissues of the
tendon reflex muscles (the quadriceps muscle shoulder are pinched under the acromion process at
group) are innervated by the L4 nerve root via the approximately 60 degrees to120 degrees of
femoral nerve. abduction. Pain throughout abduction active range
of motion suggests acromioclavicular joint
103. A 25-year-old football player fell on his dysfunction.
shoulder vertically and violently stretched his neck
in the opposite direction. He was later diagnosed 106. A tennis player receives a surgical repair of the
with a brachial plexus injury (Erb-Duchenne annular ligament. Where should the therapist
paralysis). His arm is hanging at his side in medial expect to note the most edema?
rotation in the "waiter's tip" position. What results a. Radial ulnar joint
are expected from the neurologic examination? b. Olecranon bursa
a. Paralysis of the deltoid, triceps, wrist extensors c. Ulnohumeral joint
(long and short carpi radialis), and finger extensors
d. Lateral triangle
b. Paralysis of all intrinsic muscles of the hand,
Ratio: The lateral triangle (composed of the radial
flexors muscles (claw hand), loss of sensation over
C8-T1 dermatomes, and Horner's syndrome head, olecranon process, and lateral epicondyle) is
the most likely of the choices to exhibit joint edema.
c. Hypesthesia over C5-C6 and weakness of the Joint edema is common after a surgical procedure.
deltoid, supraspinatus and infraspinatus, biceps,
107. A patient is referred to physical therapy with a
and brachioradialis muscles
history of temporomandibular joint pain. The
d. Klumpke paralysis caused by forced therapist notices that the patient is having difficulty
hyperabduction of the arm closing his mouth against minimal resistance. With
Ratio: The mechanism of injury indicates implication this information, which of the following muscles
of the superior plexus of C5-C6, causing diffuse arm would not be a target for strengthening exercise to
weakness not fitting typical radicular presentation correct this deficit?
(involvement of one myotome). Nerve regeneration a. Medial pterygoid muscle
is still possible when only the endonurium and
b. Temporalis
capillary complex are disrupted ("a burner").
However, when the perineurium (funiculus) or c. Masseter
epinurium is disrupted (brachial neuropraxia), d. Lateral pterygoid muscle
useful regeneration is not expected. This requires
Ratio: All of the listed muscles participate in
urgent neurosurgical intervention to prevent
mandibular elevation with the exception of the
permanent neurologic deficit.
lateral pterygoid muscle. The lateral pterygoid
104. A patient presents to an outpatient physical muscle and the suprahyoid muscles participate in
therapy clinic with a severed ulnar nerve of the right mandibular depression.
upper extremity. What muscle is still active and
108. A physical therapist is asked to examine a 37-
largely responsible for the obvious hyperextension
year-old man with right-side sciatica. The therapist
at the metacarpophalangeal (MCP) joints of the
performs a passive straight leg raise test of the right
involved hand?
lower extremity with the knee and ankle in neutral
a. Dorsal interossei position. In performing this test on a patient with an
b. Volar interossei L5 disc protrusion, what is the lowest degree at
which the therapist would expect to reproduce the
c. Extensor carpi radialis brevis
patient's symptoms?
d. Extensor digitorum
a. At 0 degrees of hip flexion
Ratio: An ulnar nerve-compromised hand presents
as a "claw" hand after a prolonged amount of time b. At 35 degrees of hip flexion
because of atrophy of the interossei. The extensor c. At 70 degrees of hip flexion
digitorum takes over and pulls the MCPs in d. At 90 degrees of hip flexion
hyperextension.
Ratio: During a unilateral straight leg raise of the
105. A patient presents to an outpatient clinic with involved lower extremity, tension is placed on the
complaints of shoulder pain. The therapist observes sciatic nerve at approximately 35 degrees of hip
flexion. At 0 degrees of hip flexion, tension is Ratio: The ipsilateral inferior articular process is
minimal to none, and tension is maximal above 70 seen as the front leg.
degrees of hip flexion.
Siegelman Online Advantage - HNBS,
109. Which of the following muscles would you not
expect to be affected by a C6-C7 lesion?
UE, LE (APK + SPT Questions)
1. Following examination and evaluation, a therapist
a. Biceps brachii
determines that a patient would benefit from
b. Anterior deltoid distraction of the right temporomandibular (TM)
c. Infraspinatus joint. This maneuver is accomplished by moving the
d. Triceps brachii mandible in which direction?

Ratio: Choices A, B, and C all receive innervation a. inferior glide.


from that branch of the brachial plexus. The triceps b. posterior glide.
brachii is innervated by C7-C8. c. medial glide.
110. Your patient presents with a nerve injury that d. anterior glide.
causes the thenar eminence to be flattened because
Ratio: By definition, joint distraction describes
of muscle atrophy. The thumb is adducted and
moving one joint surface perpendicular to the other
extended. You would want to test the muscles
joint surface. Since the maxillary surface of the TM
supplied by the
joint faces caudally a perpendicular movement from
a. Ulnar nerve this surface would entail an inferior glide of the TM
b. Antebrachial nerve joints condylar surface.
c. Median nerve 2. A patient presents with complaint of shooting
d. Radial nerve pain in the left lateral thigh and leg as well as
muscle weakness in the left gluteus medius and
Ratio: The ape or simian hand described is indicative peroneal (fibular) muscles. These findings are most
of a median nerve palsy. consistent with a neurologic radiculopathy at the
111. Your patient is a framer on a construction dermatomal level of:
project. He has been wearing a heavy carpenter's a. L3.
belt for the last month. He now complains of painful
hyperthesias on the proximal anterior lateral thigh. b. L5.
He gets relief with sitting, and walking seems to c. L4.
aggravate his symptoms. The structure most likely d. S1.
producing these symptoms is the
Ratio: The L5 dermatome travels down the lateral
a. Lateral femoral cutaneous nerve thigh and leg so any pain related to a neurologic
b. Motor branch of the femoral nerve condition involving the L5 spinal nerve will follow
c. Medial femoral cutaneous nerve that path. Both the gluteus medius and peroneal
muscles receive significant contributions from the
d. Inguinal nerve L5 spinal nerve, so if there was abnormal function of
Ratio: This case represents a classic presentation of L5, both of those muscles would demonstrate
meralgia parasthetica, which involves the lateral weakness.
femoral cutaneous nerve. The other choices are not 3. A patient recovering from stroke is able to
in the area. generate a contraction in the elbow flexors of the
112. Your patient complains of neck pain and affected right upper extremity but the contraction
peripheral symptoms. Radiographs revealed fades out quickly. He is unable to generate
narrowing of the C4-C5 intervertebral foramen. The significant active tension from mid-range to the
nerve root most likely involved would be the maximally shortened position. The therapist
a. C5 nerve root documents this finding as:
b. C4 nerve root a. passive insufficiency.
c. C6 nerve root b. muscle inhibition.
d. Sensory branch of C4 c. active insufficiency.
Ratio: It is well known that the C5 nerve root exits d. overload.
the C4-C5 intervertebral space. The other choices Ratio: Active insufficiency refers to the inability of a
exit above and below this level. muscle to generate significant active tension when
113. The front leg of the celebrated "Scotty Dog" is it is maximally shortened. Muscles that that cross
what anatomic structure? more than one joint are more likely to exhibit active
insufficiency. Passive insufficiency refers to the
a. Inferior articular process
inability of the muscle-tendon unit to allow the
b. Superior articular process joints to move through the full available range of
c. Pars interarticularis motion due to restrictions in length (e.g.,
contracture). Muscle inhibition results from spinal
d. Transverse process
reflex circuits (autogenic inhibition results from
activation of the Golgi tendon organ in the agonist c. flexion, side bending to the right, and rotation to
muscle; reciprocal inhibition results from activation the left.
of the muscle spindle afferents, IA, of the antagonist d. extension, side bending to the right, and rotation
muscle). Overload refers to the minimum threshold to the right.
for the intensity and duration of stress for a muscle
Ratio: During a voluntary contraction of the left
to become stronger.
multifidus, the trunk would move into extension,
4. A patient presents with abnormal neurologic into left side bending and towards right rotation. To
findings of the upper limb and the physical therapist place this muscle in its optimally lengthened
is trying to differentiate between involvement of position it should be taken into the opposite
the C8 spinal nerve and the ulnar nerve. Muscle directions in all 3 planes from the voluntary
weakness in which muscles would lead the therapist contraction.
to believe the primary dysfunction is related only to
the C8 spinal nerve? 7. A 65 years old patient is 3 months post right total
hip arthroplasty. The patient is currently ambulating
a. lumbricales III & IV. with a small base quad cane. The therapist is
b. palmaris brevis. completing an observational gait analysis and
c. flexor digitorum profundus I & II. notices a lean of the trunk over the right extremity
during stance. A possible cause for this deviation is:
d. flexor pollicis brevis (deep head).
a. iliopsoas contracture.
Ratio: The flexor digitorum profundus I & II is
innervated by C8, but not by the ulnar nerve (it is b. weak gluteus maximus.
innervated by the median nerve). The other muscles c. weak gluteus medius.
are all innervated by both the C8 spinal nerve and d. weak quadriceps.
the ulnar nerve. Therefore, the flexor digitorum
profundus I & II is the best muscle (of the ones Ratio: Lateral trunk lean is the result of a weak or
paralyzed gluteus medius on the stance leg. Lateral
listed) to differentiate between involvement of the
C8 spinal nerve versus the ulnar nerve. lean can also occur with a painful hip as the patient
leans to reduce forces on the hip. Excessive hip
5. The physical therapist's examination of a 9 week- flexion is the result of hip and/or knee flexion
old infant with a diagnosis of Erb's palsy reveals contractures. Backward trunk lean is the result or
muscle weakness in the shoulder and elbow. The weakness or paralysis of the gluteus maximus on
portion of the brachial plexus that is affected would the stance leg or an anteriorly rotated pelvis.
be the: Forward trunk lean is the result of quadriceps
a. middle root: C7. weakness or hip and knee flexion contractures.
b. upper roots: C5-C6. 8. The patient presents with a diagnosis of shoulder
c. lower roots: C8-T1. dysfunction. Tests and measures indicate a positive
anterior apprehension sign. This most likely is
d. upper and middle root C5-C7. representative of:
Ratio: Injury to upper roots (the most common a. rotator cuff impingement.
injury) is known as Erb's palsy. Erb's palsy (paralysis)
affects the shoulder and elbow, because the upper b. Grade I acromioclavicular sprain.
roots supply these structures. Injury to the lower c. subacromial bursitis.
roots is known as Klumpke's paralysis, but is d. past history of anterior shoulder dislocation.
relatively uncommon, and very rarely exists by
itself. Because the lower roots are injured, the hand Ratio: The anterior apprehension sign specifically
demonstrates a history of previous anterior
is predominantly affected. If the lower roots are
injured, the injury is generally so severe that all the instability of the glenohumeral joint most
commonly related to a traumatic dislocation of the
roots of the plexus are involved, and the injury
involves all parts of the arm. glenohumeral joint. The patient is supine and the
extremity is moved into abduction and external
6. A patient presents with low back pain. Following rotation. The patient will not allow the motion or
examination and evaluation the physical therapist expresses extreme concern. While the other
chooses to perform soft tissue techniques as a conditions mentioned might elicit some pain when
component of the patient's plan of care. While moved into this position, the apprehensiveness is
performing these techniques the therapist decides usually reserved for the possibility of dislocating the
to take the left multifidus into its maximally shoulder during the test.
lengthened position to provide a passive stretch.
Which positions of the trunk would maximally Dunaway A (AnaKines Questions)
lengthen the patient's left multifidus? 1.While assessing the standing posture of a patient,
a. extension, side bending to the left, and rotation the therapist notes that a spinous process in the
to the left. thoracic region is shifted laterally. The therapist
estimates that T2 is the involved vertebra because
b. flexion, side bending to the left, and rotation to
he or she notes that it is at the approximate level of
the right.
the 4
A. inferior angle of the scapula D. Extensor digitorum
B. superior angle of the scapula 7. A physician notes a vertebral fracture in the X-ray
C. spine of the scapula of a patient involved in a car accident. The fractured
vertebra has a bifid spinous process. Which of the
D. xiphoid process of the sternum following vertebrae is the most likely to be
2. The therapist is evaluating a 32-year-old woman involved? 40
for complaints of right hip pain. The patient has A. Fourth lumbar vertebra
injured the strongest ligament of the hip. The
therapist places the patient in the prone position on B. Fifth cervical vertebra
the plinth and passively extends the involved hip. C. Twelfth thoracic vertebra
The therapist notes an abnormal amount of D. First sacral vertebra.
increase in passive hip extension. Which of the
8. A patient presents to an outpatient facility with
following ligaments is damaged? 14
complaints of pain in the groin area (along the
A. Ischiofemoral ligament medial left thigh). With manual muscle testing of
B. Iliofemoral ligament (Y ligament of Bigelow) the involved lower extremity a therapist determines
C. Pubofemoral ligament the following: hip flexion = 4+/5, hip extension =
4+/5, hip abduction = 4+/5, hip adduction = 2+/5,
D. Ligamentum teres hip internal rotation = 2+/5, and hip external
3. Which of the following actions places the greatest rotation = 2+/5. Which nerve on the involved side is
stress on the patellofemoral joint? 18 most likely injured? 44
A. When the foot first contacts the ground during A. Lateral cutaneous nerve of the upper thigh.
the gait cycle. B. Obturator nerve
B. Exercising on a stair stepper machine. C. Femoral nerve.
C. Running down a smooth decline of 30 o D. Ilioinguinal nerve.
D. Squats to 120 o of knee flexion. 9. A teenager comes to an outpatient facility with
4. The therapist is treating a track athlete who complaints of pain at the tibial tubercle when
specializes in sprinting and wants to increase his or playing basketball. The therapist notices that the
her speed on the track. To accomplish this goal, the tubercles are abnormally pronounced on bilateral
plan of care should include activities to develop knees. What condition does the patient most likely
fast-twitch muscle fibers. Characteristics of this type have? 58
of fiber include: 27 A. Jumper’s knees.
A. Fatigues slowly, fiber colors appear red and used B. Anterior cruciate ligament sprain.
more in aerobic activity.
C. Osgood-Schlatter disease.
B. Fatigues quickly, fiber colors appear white and
used in anaerobic activity. D. A & C
C. Fatigues quickly, fiber colors appear white and
used more in aerobic activity. 10. Which of the following is observed by the
D. Fatigues slowly, fiber colors appear white, and therapist if a patient is correctly performing an
used more in anaerobic activity. anterior pelvic tilt in standing position? 61
5. A physical therapist should place the knee in A. Hip extension and lumbar flexion.
which of the following positions to palpate the B. Hip flexion and lumbar extension.
lateral collateral ligament (LCL)? 28 C. Hip flexion and lumbar flexion.
A. Knee at 60 o of flexion and the hip externally D. Hip extension and lumbar extension.
rotated.
11. If the line of gravity is posterior to the hip joint
B. Knee at 20 o of flexion and the hip at neutral. in standing, on what does the body first rely to keep
C. Knee at 90 o of flexion and hip externally rotated. the trunk from moving into excessive lumbar
D. Knee at 0 o and the hip at neutral. extension? 63
A. Iliopsoas muscle activity
6. A patient presents to an outpatient physical B. Abdominal muscle activity
therapy clinic with a severed ulnar nerve in the C. Anterior pelvic ligaments and the hip joint
upper extremity. What muscle is still active and capsule.
largely responsible for the obvious hyperextension D. Posterior pelvic ligaments and the hip joint
at the metacarpophalangeal (MCP) joints of the capsule.
involved hand? 39
12. The therapist receives a referral to evaluate a
A. Dorsal interossei patient with a boutonniere deformity. With this
B. Volar interossei injury, the involved finger usually presents in the
C. Extensor carpi radialis brevis position of 64
A. Flexion of the proximal interphalangeal (PIP) joint A. Full abduction, full flexion and full external
and flexion of the distal interphalangeal (DIP) joint. rotation
B. Extension of the PIP joint and flexion of the DIP B. Full abduction, full flexion and full internal
joint. rotation
C. Flexion of the PIP joint and extension of the DIP C. Full adduction, full external rotation, and full
joint. extension
D. Extension of the PIP joint and extension of the D. Full adduction, full internal rotation and full
DIP joint. extension
13. What is the closed-packed position of the 19. What ligament is most involved in sustaining the
shoulder? 78 longitudinal arch of the foot? 102
A. Internal rotation and abduction A. Plantar calcaneonavicular ligament
B. External rotation and abduction B. Long plantar ligament
C. Internal rotation and adduction C. Plantar calcaneocuboid ligament
D. External rotation and adduction D. Anterior talofibular ligament
14. A 42-year-old receptionist presents to an 20. A posterior lateral herniation of the lumbar disc
outpatient physical therapy clinic complaining of between vertebrae L4 and L5 most likely results in
low back pain. The therapist decides that postural damage to which nerve root? 103
modification needs to be part of the treatment plan. A. L4
What is the best position for the lower extremities
B. L5
while the patient is sitting? 79
A. 90 o of hip flexion, 90 o of knee flexion, and 10 o C. L4 and L5
of dorsiflexion D. L5 and S1
B. 60 o of hip flexion, 90 o of knee flexion, and 0 o 21. Which of the following is not an example of a
of dorsiflexion synarthrodial joint in the body? 120
C. 110 o of hip flexion, 80 o of knee flexion, and 10 o A. Coronal suture
of dorsiflexion B. The fibrous joint between the shaft of the tibia
D. 90 o of hip flexion, 90 o of knee flexion, and 0 o and fibula
of dorsiflexion. C. Symphysis pubis
15. A patient with a diagnosis of a rotator cuff tear D. Metacarpophalangeal
has just begun active range of motion. The therapist
22. In the geriatric population, _________usually
is strengthening the rotator cuff muscles to increase
occurs after _______ is present. 121
joint stability and oppose the superior shear of the
deltoid. Which of the rotator cuff muscles A. Spondylolisthesis, spondylolysis
participate least in opposing the superior shear B. Spondylolysis, spondylolisthesis
force of the deltoid? 86 C. Spondyloschisis, spondylolysis
A. Infraspinatus D. Spondylolisthesis, spondyloschisis
B. Subscapularis 23. A 67-year-old woman presents to an outpatient
C. Teres minor facility with a diagnosis of right adhesive capsulitis.
D. Supraspinatus The therapist plans to focus mostly on gaining
abduction range of motion. In which direction
16. What portion of the adult knee meniscus is
should the therapist mobilize the shoulder to gain
vascularized? 88
abduction range of motion? 122
A. Outer edges
A. Posteriorly
B. Inner edges
B. Anteriorly
C. The entire meniscus is vascular
C. Inferiorly
D. The entire meniscus is avascular
D. Superiorly
17. What is the most likely cause of anterior pelvic
24. A 13-year-old girl has fractured the left patella
tilt during initial contact (heel strike)? 90
during a volleyball game. The physician determines
A. Weak abdominals that the superior pole is the location of the fracture.
B. Tight hamstrings Which of the following should be avoided in early
rehabilitation? 123
C. Weak abductors
D. Back pain A. Full knee extension
B. 45 o of knee flexion
18. In what position should the therapist place the
upper extremity to palpate the supraspinatus C. 90 o of knee flexion
tendon? 101 D. 15 o of knee flexion
25. A football player presents to an outpatient clinic C. Deep peroneal
with complaints of pain in the right knee after an D. Anterior tibial
injury suffered the night before. The physician
31. A patient is sitting over the edge of a table and
determines that the anterior cruciate ligament (ACL)
is torn. Which of the following is most commonly performing active knee extension exercises using an
ankle weight as resistance. This exercise
associated with an injury causing damage to the ACL
only? 125 demonstrates what class lever? 147
A. First class
A. Varus blow to the knee with the foot planted and
an audible pop B. Second class
B. Foot planted, medial tibial rotation, and an C. Third class
audible pop. D. Fourth class
C. Valgus blow to the knee with the foot planted 32. A therapist is evaluating the gait pattern of a
and no audible pop. patient and notes that the pelvis drops inferiorly on
D. Foot planted, lateral tibial rotation, and no the right during the midswing phase of the right
audible pop. lower extremity. The patient also leans laterally to
the left with the upper trunk during this phase.
26. A patient presents to an outpatient clinic with
complaints of shoulder pain. The therapist observes Which of the following is the most likely cause of
this deviation? 151
a painful arc between 70 o and 120 o of active
abduction in the involved shoulder. This finding is A. Weak right gluteus medius
most indicative of what shoulder pathology? 127 B. Weak right adductor longus
A. Rotator cuff tear C. Weak left gluteus medius
B. Acromioclavicular joint separation D. Weak left adductor longus
C. Impingement 33. The therapist is performing an orthopedic test
D. Labrum tear that involves: (1) placing the patient in a sidelying
27. A tennis player receives a surgical repair of the position, (2) placing the superior lower extremity in
hip extension and hip abduction, (3) placing the
annular ligament. Where should the therapist
expect to note the most edema? 129 knee of the superior lower extremity in 90 o of
flexion, and (4) allowing the superior lower
A. Radial ulnar joint extremity to drop into adduction. Failure of the
B. Olecranon bursa superior lower extremity to drop indicates a tight:
C. Ulnohumeral joint 152
D. Lateral triangle A. iliopsoas
28. In developing the plan of care for a 28-year-old B. rectus femoris
pregnant woman which of the following muscles C. iliotibial band
should be the focus of the strengthening exercises
to maintain a strong pelvic floor? 135 D. hamstring
34. A therapist assessing radial deviation range of
A. Piriformis, obturator internus, and
pubococcygeus motion at the wrist. The correct position of the
goniometer should be as follows: the proximal arm
B. Obturator internus, pubococcygeus and aligned with the forearm and the distal arm aligned
coccygeus with the third metacarpal. What should be used as
C. Rectus abdominis, iliococcygeus, and piriformis the axis point? 172
D. Iliococcygeus, pubococcygeus, and coccygeus A. Lunate
29. The therapist is performing an orthopedic test B. Scaphoid
on a 25-year-lld man with the chief complaint of low C. Capitate
back pain. The patient has a positive Thomas test.
With this information, what might the therapist D. Triquetrum
need to include in the treatment plan? 138 35. Observing a patient in a standing position, the
A. Stretching of the hip abductors therapist notes that an angulation deformity of the
right knee causes it to be located medially in
B. Stretching of the hip adductors relation to the left hip and foot. This condition is
C. Stretching of the hip extensors commonly referred to as: 179
D. Stretching of the hip flexors A. genu varum
30. A patient who has suffered a recent fracture of B. genu valgum
the right tibia and fibula has developed foot drop of C. pes cavus
the right foot during gait. Which nerve is causing
D. none of the above
this loss of motor function? 144
A. Posterior tibial 36. Which of the following is the most vulnerable
position for dislocation of the hip? 180
B. Superficial peroneal
A. 30 o hip extension, 30 o hip adduction, and this information, which of the following muscles
minimal internal rotation would not be a target for strengthening exercise to
B. 30 o hip flexion, 30 o hip adduction, and minimal correct this deficit? 64
external rotation A. Medial pterygoid muscle
C. 30 o hip flexion, 30 o hip abduction, and minimal B. Temporalis
external rotation. C. Masseter
D. 30 o hip extension, 30 o hip abduction, and D. Lateral pterygoid muscle
minimal external rotation.
4. A 65-year-old man presents to physical therapy
37. Which of the following articulate with the with complaints of pain due to compression
second cuneiform? 181 fractures of the C2 and C3 vertebrae. The patient
A. Navicular has an unusually large cranium. He describes his
B. Talus condition by stating, “Much of my bone tissue is
continually decreasing, then reforming”. The patient
C. First metatarsal also indicates that the condition has caused limb
D. Cuboid deformity. Which of the following diseases does he
have? 95
38. A patient has traumatically dislocated the tibia A. Paget’s disease
directly posteriorly during an automobile accident. B. Achondroplastic dwarfism
Which of the following structures is the least likely C. Osteogenesis imperfecta
to be injured? 196
D. Osteopetrosis
A. Tibial nerve
5. A therapist is assisting a patient in gaining lateral
B. Popliteal artery stability of the knee joint. The therapist is using
C. Common peroneal nerve strengthening exercises to strengthen muscle
D. Anterior cruciate ligament groups that will increase active restraint on the
lateral side of the joint. Which of the following
Dunaway B (AnaKines Questions) offers the least amount of active lateral restraint?
1. A therapist places a pen in front of a patient and 113
asks him to pick it up and hold it as he normally A. Gastrocnemius
would to write. The patient picks the pen up and
B. Popliteus
holds it between the pad of the thumb and the
middle and index fingers. Why type of grasp or C. Biceps femoris
prehension is the patient using? 44 D. Iliotibial band
A. Palmar prehension 6. A clinical instructor is explaining to a physical
B. Fingertip prehension therapy student the function of the screw home
mechanism in the knee joint. Part of the therapist’s
C. Lateral prehension
explanation involves teaching the student the
D. Hook grasp movement of the tibia and femur during closed
2. A therapist is assessing a patient in an attempt to chain activities. When the knee joint is extended in
discover the source of her pain. She positions the a closed-chain activity, which of the following
patient’s cervical spine in different directions in an statements is true? 114
attempt to elicit the patient’s symptoms. In one A. The femur laterally rotates on the tibia.
such direction, the patient reports return of
B. The femur medially rotates on the tibia
symptoms, including pain located at the right
posterior scapular region, which extends down the C. The tibia laterally rotates on the femur
posterior side of the right upper extremity to the D. The tibia medially rotates on the femur
ends of the fingers, and tingling in the second, third,
7. A patient is in prone position with his head
and fourth digits. The patient also indicates that she
rotated to the left side. The left upper extremity is
often has decrease in sensation on the dorsal side of
placed at his side and fully internally rotated. The
the second and third digits. She also has noticeable
left shoulder is then shrugged toward the chin. The
weakness in the right triceps. Which nerve root is
therapist then grasps the midshaft of the patient’s
most likely involved? 50
left forearm. The patient is then instructed to “try to
A. Fourth cervical root reach your feet using just your left arm.” This
B. Fifth cervical root movement is resisted by the therapist. The test is
assessing the strength of what muscle? 117
C. Sixth cervical root
A. Upper trapezius
D. Seventh cervical root
B. Posterior deltoid
3. A patient is referred to physical therapy with a
history of temporomandibular joint pain. The C. Latissimus dorsi
therapist notices that the patient is having difficulty D. Triceps brachii
closing his mouth against minimal resistance. With
8. A physical therapist is beginning an evaluation of A. Patellofemoral joint
a patient with a diagnosis of “knee strain”. Range of B. Anterior cruciate ligament
motion limitation does not follow the normal
C. Medial meniscus
capsular pattern of the knee. Which of the following
are possible causes of the restriction in range of D. Femoral condyles
motion? 132 13. A 14-year-old girl presents to an outpatient
A. Ligamentous adhesions physical therapy clinic with complaints of anterior
knee pain for 2 weeks. The patient notes no
B. Internal derangement
particular incident of onset. She indicates that pain
C. Extra-articular lesions increases when she attempts to ascend and
D. All of the above descend stairs and with squatting to 130 o of knee
9. In taping an athlete’s ankle prophylactically flexion. The evaluation show limited quadriceps
before a football game, in what position should the strength on the involved side at 4/5 with manual
ankle be slightly positioned before taping to provide muscle testing and normal hamstring strength. All
the most protection against an ankle sprain? 138 meniscus and ligament tests are negative. Given the
above information, what is the most likely cause of
A. Inversion, dorsiflexion, abduction the signs and symptoms? 150
B. Eversion, plantarflexion, adduction A. Lateral glide of the patella
C. Eversion, dorsiflexion, abduction B. Medial tilt of the patella
D. Inversion, plantarflexion, adduction C. Baker’s cyst
10. A high school athlete is considering whether to D. Anterior cruciate ligament tear
have an anterior cruciate ligament reconstruction.
The therapist explains the importance of this 14. A therapist is evaluating a patient who
ligament, especially in a person that is young and complains of frequent foot, ankle and knee pain.
athletic. Which of the statements below is correct in The therapist asks the patient to assume a standing
describing part of the function of the anterior position with the knees slightly flexed. The therapist
cruciate ligament? 146 then demonstrates active bilateral foot pronation to
the patient. When asked to perform this task, the
A. The anterior cruciate ligament prevents excessive patient has difficulty. Which of the following
posterior roll of the femoral condyles during flexion limitations is a possible cause of the patient’s
of the femur at the knee joint difficulty in performing this task? 154
B. The anterior cruciate ligament prevents excessive A. Restriction limiting plantar flexion and lateral
anterior roll of the femoral condyles during flexion rotation of the talus
of the femur at the knee joint
B. Restriction limiting dorsiflexion and medial
C. The anterior cruciate ligament prevents excessive rotation of the talus
posterior roll of the femoral condyles during
extension of the femur at the knee joint C. Restriction limiting eversion of the calcaneus and
medial rotation of the talus
D. The anterior cruciate ligament prevents excessive
anterior roll of the femoral condyles during D. B and C are correct
extension of the femur at the knee joint 15. A physical therapist is evaluating a female
11. A therapist is evaluating a patient in the distance runner who complains of intermittent
intensive care unit. While performing a chart medial ankle pain. In static standing, the therapist
review, the therapist discovers that the patient was palpates excessive lateral deviation of the head of
seriously injured to a fall from a 3-story building. the talus. From this information, in what position is
The therapist determines from the physician’s the subtalar joint during palpation? 157
evaluation that of the many injuries the patient has A. Supination
sustained, a rupture of two ligaments extends from B. Pronation
the side of the dens to the medial side of the
C. Neutral
occipital condyle. Which of the following was
injured? 147 D. Unable to determine from the information given
A. Ligamentum nuchae 16. A patient presents to therapy with an ankle
B. Tectorial membrane injury. The therapist has determined that the injury
is at the junction of the distal tibia and fibula. Which
C. Posterior atlanto-occipital ligament of the following functions most in preventing
D. Alar ligament excessive external rotation and posterior
12. A patient is an outpatient facility because of an displacement of the fibula? 158
injury sustained to the right knee joint. Only the A. Anterior inferior tibiofibular ligament
structures within the synovial cavity were B. Posterior inferior tibiofibular ligament
compromised during the injury. Knowing this
C. Interosseous membrane
information only, the therapist is not concerned
with injury to which of the following structure? 149 D. None of the above
17. A 17-year-old athlete has just received a D. Median nerve – pronator teres
posterior cruciate ligament reconstruction. The 23. A patient presents to outpatient physical
therapist is attempting to explain some of the therapy with tarsal tunnel syndrome. What nerve is
characteristics of the posterior cruciate ligament. involved? Where should the therapist concentrate
Which of the following is incorrect information? 161 treatment? 177
A. The posterior cruciate ligament prevents A. Superficial peroneal nerve – inferior to the
posterior translation of the tibia on the femur. medial malleolus
B. Posterior bands of the posterior cruciate ligament B. Posterior tibial nerve – inferior to the medial
are their tightest in full knee extension. malleolus
C. The posterior cruciate ligament is attached to the C. Superficial peroneal nerve – inferior to the lateral
lateral meniscus and not to the medial meniscus malleolus
D. The posterior cruciate ligament helps with medial D. Posterior tibial nerve – inferior to the lateral
rotation of the tibia during full knee extension with malleolus.
open
24. When ambulating on uneven terrain, how
chain activities. should the subtalar joint be positioned to allow
18. A 14-year old girl place excessive valgus stress to forefoot rotational compensation? 184
the right elbow during a fall from a bicycle. Her A. Pronation
forearm was in supination at the moment the valgus
stress was applied. Which of the following is most B. Supination
likely involved in this type of injury? 165 C. Neutral position
A. Ulnar nerve D. The position of the subtalar joint does not
influence forefoot compensation
B. Extensor carpi radialis
C. Brachioradialis Dutton (AnaKines Questions)
D. Annular ligament 1. A patient is referred to physical therapy for
stretching of the right sternocleidomastoid muscle.
19. Which tendon is most commonly involved with
The MOST effective method to stretch the muscle is
lateral epicondylitis? 166
by positioning the head and neck into:
A. Extensor carpi radialis longus
A.extension, left side-bending, and right rotation.
B. Extensor carpi radialis brevis
B.flexion, left side-bending, and left rotation.
C. Brachioradialis
C. extension, right side-bending, and left rotation.
D. Extensor digitorum
D. flexion, right side-bending, and left rotation.
20. Which of the following is not part of the
Correct Answer: A
triangular fibrocartilage complex of the wrist? 168
Rationale: The right sternocleidomastoid muscle
A. Dorsal radioulnar ligament
produces flexion right side bending and left rotation
B. Ulnar collateral ligament -- it should therefore be stretched in the opposite
C. Radial collateral ligament directions.
D. Ulnar articular cartilage 2. You are preparing to perform joint mobilizations
21. Which of the following muscle tendons most on a patient’s proximal radioulnar joint. How would
commonly sublux in patients who suffer from you position thejoint to place it in the loose packed
rheumatoid arthritis? 172 position?
A. Flexor digitorum profundus A. 10 degrees flexion, 10 degrees pronation
B. Extensor carpi ulnaris B. 20 degrees flexion, 45 degrees supination
C. Extensor carpi radialis longus C. 45 degrees flexion, 20 degrees supination
D. Flexor pollicis longus D. 70 degrees flexion, 35 degrees supination
22. A patient presents to an outpatient clinic with Correct Answer: D
an order to evaluate and treat the right forearm and Rationale: This is the loose pack position for the
wrist secondary to nerve compression. The patient proximal radioulnar joint.
has the following signs and symptoms: pain with 3. A patient is referred to physical therapy with a
manual muscle testing of pronation, decreased history of temporomandibular joint pain and
strength of the flexor pollicis longus and pronator difficulty closing her mouth against minimal
quadratus, and pain with palpation of the pronator resistance. Which of the following muscles would
teres. What nerve is most likely compromised? not be a target for strengthening to help close the
What is the most likely area of compression? 174 mouth?
A. Median nerve – carpal tunnel A. Lateral pterygoid
B. Ulnar nerve – Guyon’s canal B. Masseter
C. Ulnar nerve – pronator quadratus C. Medial pterygoid
D. Temporalis A. Axillary nerve
Correct Answer: A B. Musculocutaneous nerve
Rationale: The lateral pterygoid opens the mouth. C. Dorsal scapular nerve
4.You are treating a patient for generalized D. Long thoracic nerve
weakness following a prolonged period on bedrest. Correct Answer: A
The patient is able to ambulate well on the level,
but is having difficulty with ascending stairs. Rationale: The accessory nerve is commonly injured
with a fracture of the surgical neck of the humerus.
Although he is able to position his foot on the step,
he is unable to climb the stairs without pulling hard 8.You are treating a patient who sustained an injury
on the railing. Which of the following muscles will to the area of the wrist known as the anatomical
need to be strengthened to help this patient climb snuff-box. Which of the following structures do not
the stairs? pass through the snuff-box?
A. Hip and knee extensors A. Extensor pollicis longus
B. Biceps and triceps of both upper extremities B. Abductor pollicis brevis
C. Hip extensors and core muscles C. Extensor pollicis brevis
D. Hip and knee flexors D. Abductor pollicis longus
Correct Answer: A Correct Answer: D
Rationale: Hip and knee extension strength is Rationale: The abductor pollicis longus does not
necessary for ascending stairs, which involves the pass through the anatomical snuff-box.
patient’s ability to straighten the knee and then 9.You are designing a strengthening program for a
extend the hip. patient with weak hip muscles. Each of the
5.You are trying to teach a 15-year-old patient to following muscles can produce movement of the
use crutches and non-weightbearing on the right, hip, except:
but the patient is having difficulty due to weakness A. Rectus femoris
of the upper quadrant muscles. Which of the
B. Obturator internus
following muscles would be the most important to
strengthen? C. Quadratus femoris
A. deltoid, biceps, and brachialis D. Vastus medialis
B. lower trapezius, latissimus dorsi, and triceps Correct Answer: D
C. upper trapezius, rhomboids and serratus anterior Rationale: The vastus medialis does not cross the
hip joint.
D. deltoid, upper trapezius and levator scapulae
10.You are applying massage to a muscle within the
Correct Answer: B
femoral triangle. Which of the following muscles is
Rationale: All of these muscles are involved when not found in the femoral triangle?
using crutches.
A. Adductor magnus
6. You are treating a 25-year-old female athlete
B. Adductor longus
with a diagnosis of patellofemoral syndrome. As
part of the patient’s intervention you decide to use C. Pectineus
both open and closed kinetic chain exercises. Which D. Sartorius
of the following exercises places the most stress on Correct Answer: A
the patellofemoral joint?
Rationale: The adductor magnus is not found within
A. Closed kinetic chain exercises involving 90 the femoral triangle.
degrees of knee flexion
11.You are designing a strengthening program for a
B. Open kinetic chain exercises performed between patient recovering from an anterior shoulder
25 and 90 degrees of knee flexion dislocation. Which of the following muscle groups
C. Open kinetic chain exercises performed between should be emphasized?
120 and 90 degrees of knee flexion A. Shoulder adductors and internal rotators
D. Closed kinetic chain exercises involving greater B. Shoulder abductors and external rotators
than 120 degrees of knee flexion
C. Shoulder adductors and external rotators
Correct Answer: D
D. Shoulder abductors and internal rotators
Rationale: Closed kinetic chain exercises involving
greater than 120° of knee flexion place a very Correct Answer: A
significant amount of force through the Rationale: The shoulder adductors and internal
patellofemoral joint. rotators provide support for the anterior joint
7. You are treating a patient who is recovering from capsule.
a fracture of the surgical neck of the humerus, 12.You are treating a patient who traumatically
resulting in atrophy of the deltoid muscle. Which dislocated the tibia directly posteriorly. Which of
nerve was likely damaged by the fracture?
the following structures is the least likely to be Rationale: The teres major is considered part of the
injured? rotator cuff. It is the teres minor that is part of the
A. Tibial nerve rotator cuff.
B. Common fibularis (peroneal) nerve 17.During pregnancy, the presence of the hormone
relaxin can lead to abnormal joint hypermobility
C. Anterior cruciate ligament and pain, MOST frequently affecting the:
D. Popliteal artery A. Sacroiliac joints
Correct Answer: B B. Shoulder joints
Rationale: The common fibularis (peroneal) nerve C. Joints of the hands and feet
travels over the lateral aspect of the knee and is
therefore the least likely to be injured. D. Temporomandibular joints
13.You have been asked to tape an athlete’s ankle Correct Answer: A
prophylactically before a basketball game to Rationale: The sacroiliac joint is the most affected
prevent ankle sprain. In what position should the due to its heavy reliance on ligamentous support.
ankle be slightly positioned in before taping to 18.All of the following are considered to be the
provide the most protection? correct open-packed position of the joint, except:
A. Inversion, plantarflexion, adduction A. Ulnohumeral:70 degrees of flexion, 10 degrees of
B. Eversion, dorsiflexion, abduction supination
C. Eversion, plantarflexion, adduction B. Radiohumeral: 70 degrees of flexion, 35 degrees
D. Inversion, dorsiflexion, abduction of supination
Correct Answer: D C. Glenohumeral: 55 degrees of abduction, 30
degrees of horizontal adduction
Rationale: This position limits inversion,
plantarflexion, and adduction. D. Distal radioulnar: 10 degrees of supination
14. You have been asked to fabricate a splint for a Correct Answer: B
one-month-old infant with congenital hip Rationale: The open pack position for the
dislocation. In what position should the hip be radiohumeral joint is full elbow extension, full
replaced while in the splint? forearm supination.
A. Flexion and abduction 19.You are assessing a patient’s arm and shoulder
B. Flexion and adduction strength. When assessing internal rotation at the
shoulder, which of the following muscles would not
C. Extension and adduction be involved?
D. Extension and abduction A. Pectoralis major
Correct Answer: A B. Infraspinatus
Rationale: This is the most stable position. C. Latissimus dorsi
15.Which of the following structures provide D. Subscapularis
anterolateral stability to the knee?
Correct Answer: B
A. Anterior cruciate ligament, lateral collateral
ligament, and iliotibial band Rationale: The infraspinatus is an external rotator of
the shoulder.
B. Posterior cruciate ligament, medial collateral
ligament, and biceps femoris tendon 20. You are performing a manual muscle test of a
patient’s hip. The patient reports pain with resisted
C. Anterior cruciate ligament, medial collateral hip external rotation. Which of the following
ligament, and gastrocnemius muscles are not involved with hip external rotation?
D. Posterior cruciate ligament, lateral collateral A. Sartorius
ligament, and lateral meniscus
B. Pectineus
Correct Answer: A
C. Quadratus femoris
Rationale: The anterior cruciate ligament, lateral
collateral ligament, and iliotibial band provide D. Gluteus minimus
anterolateral stability to the knee. Correct Answer: D
16.All of the following are considered part of the Rationale: The gluteus minimus is an internal
rotator cuff, except: rotator of the hip.
A. Infraspinatus 21. You are assessing the strength of the foot and
B. Teres major ankle muscles. Which of these muscles would be
tested when resistance is applied toward
C. Subscapularis plantarflexion and eversion.
D. Supraspinatus A. Tibialis anterior
Correct Answer: B B. Tibialis posterior
C. Fibularis (peroneus) longus C. lateral (femoral) cutaneous nerve of the thigh
D. Fibularis (peroneus) brevis D. obturator
Correct Answer: A Correct Answer: C
Rationale: The tibialis anterior functions to dorsiflex Rationale: Dysfunction of this nerve leads to a
the ankle joint and assists with inversion of the foot. condition called meralgia paresthetica.
22. You are examining a female athlete in the 27.The saphenous nerve supplies cutaneous
standing position. You note that the patient’s sensation to the medial aspect of the leg. From
patella is located medially in relation to the which nerve does the saphenous nerve arise?
ipsilateral hip and ankle. This condition is commonly A. obturator
redferred to as:
B. sciatic
A. Genu valgum
C. femoral
B. Genu varus
D. it arises as a direct branch from the sacral plexus
C. Patella alta
Correct Answer: C
D. Patella baja
Rationale: The saphenous nerve is one of the
Correct Answer: A sensory branches of the femoral nerve.
Rationale: This is the correct term to describe the 28. A patient presents with a sensory deficit in an
finding. area adjacent to the dorsal surfaces of the first and
23. During the gait cycle, which of the following second toes. Which nerve might be involved?
muscles do not work eccentrically during A. deep branch of the fibular (peroneal) nerve
midstance?
B. femoral
A. Posterior tibialis
C. obturator
B. Flexor hallucis longus
D. saphenous
C. Flexor digitorum longus
Correct Answer: A
D. Gastrocnemius
Rationale: This is the area of sensory loss associated
Correct Answer: D with the deep branch of the fibular (peroneal)
Rationale: The gastrocnemius works concentrically nerve.
at midstance. 29.The anterior interosseus branch of the median
24. After a nerve injury, regeneration occurs nerve innervates which muscles?
proximally first and then progresses distally at a rate A. flexor pollicis longus
of about 1 mm per day. Following a radial nerve
injury in the axilla, which muscle would be the last B. pronator teres
to recover? C. pronator quadratus
A. long head of the triceps D. both A and C
B. extensor indicis Correct Answer: D
C. supinator Rationale: Both of these muscles are innervated by
the anterior interosseous branch of the median
D. extensor digiti minimi
nerve. The pronator teres is innervated by the
Correct Answer: B median nerve.
Rationale: Of those listed, the extensor indicis is the 30.The primary extensor of the
most proximal muscle to the axilla. metacarpophalangeal joints in the hand is the:
25. A bony mass is vaguely felt in the posterior A. extensor digitorum communis
midline of the neck about two finger breadths
B. extensor carpi radialis longus
beneath the occipital protuberance. It is the:
A. inferior articular process of C 1 C. extensor carpi ulnaris
D. dorsal interossei muscles
B. spinous process of the atlas
C. odontoid process 31. A sensory examination reveals light touch
impairment to the anterolateral thigh, lateral calf,
D. spinous process of the axis and sole the foot. When recording these findings
Correct Answer: D what are the corresponding dermatomes
Rationale: This is the location of the spinous process respectively?
of the axis. A. L2, L4, S3
26. A patient complains of a burning sensation in B. L1, L3, L5
the anterolateral aspect of the thigh. Dysfunction of C. L2, L5, S1
which nerve could lead to these symptoms?
D. L3, L5, S1
A. ilioinguinal
Correct Answer: C
B. genitofemoral
Rationale: This is the correct order for the C. Middle trunk of the brachial plexus
corresponding dermatomes. D. Posterior cord of the brachial plexus
32. An injury to the posterior cord of the brachial Correct Answer: D
plexus would most likely involve damage to which
of the following nerve(s)? Rationale: The deltoid is served by the axillary
nerve, while the wrist extensors are served by the
A. Musculocutaneous radial nerve, both of which originate from the
B. Axillary posterior cord of the brachial plexus.
C. Radial 37. A patient was involved in a motorcycle accident
D. b and c and it is suspected that he may have avulsed his C-5
nerve root at its origin. To test this impression, what
Correct Answer: D is the best muscle to check electrophysiologically?
Rationale: The posterior cord serves both the radial A. Supinator
and axillary nerves.
B. Pronator teres
33. A physical therapist observes that a patient’s
medial longitudinal arch is extremely depressed. C. Rhomboids
What ligament helps maintain the medial D. Latissimus dorsi
longitudinal arch? Correct Answer: C
A. Talonavicular Rationale: The rhomboids are served by the C4-5
B. Anterior talofibular level.
C. Plantar calcaneonavicular 38. A patient with a musculocutaneous nerve injury
D. Posterior talofibular is still able to flex the elbow. The major muscle
causing this elbow flexion is the:
Correct Answer: C
A. Brachioradialis
Rationale: The plantar calcaneonavicular ligament
helps to maintain the medial longitudinal arch of B. Flexor carpi ulnaris
the foot and by providing support to the head of the C. Pronator quadratus
talus bears the major portion of the body weight. It D. Extensor carpi ulnaris
is the main part of the spring ligament complex.
Correct Answer: A
34.When palpating the wrist and hand, which
Rationale: Of those listed, the brachioradialis is the
metacarpal should articulate with the hamate?
strongest elbow flexor and is not innervated by the
A. First musculocutaneous nerve.
B. Second 39. You are evaluating a 15-year-old female
C. Third distance runner for foot pain of unknown etiology.
D. Fourth As you palpate along the medial aspect of the foot
and ankle you palpate the head of the first
Correct Answer: D metatarsal bone and the metatarsophalangeal joint.
Rationale: The 4th metacarpal articulates with the Immediately proximal to this you identify the first
hamate. cuneiform. What large bony prominence would you
35.The Q-angle is designed as a measurement to expect to find next if you continue to move in and a
determine the amount of lateral force on the proximal direction?
patella. What three bony landmarks are used to A. Talar head
measure the Q-angle?
B. Navicular
A. Anterior superior iliac spine, superior border of
the patella, tibial tubercle C. Medial malleolus
B. Anterior superior iliac spine, midpoint of the D. Cuboid
patella, tibial tubercle Correct Answer: B
C. Anterior superior iliac spine, inferior border of Rationale: The navicular would be the next bone to
the patella, midpoint of the patella tendon be palpated proximally.
D. Greater trochanter, midpoint of the patella, 40.The patient is referred to physical therapy
superior border of the patella tendon diagnosed with pes anserine bursitis. Which muscle
Correct Answer: B does not contribute to the pes anserine?
Rationale: These are the correct landmarks to use A. Sartorius
when assessing the Q=angle. B. Semimembranosus
36. A patient presents with severe weakness of the C. Semitendinosus
deltoid muscle and wrist extensors. Where would D. Gracilis
you suspect the lesion to be located
Correct Answer: B
A. C6 Nerve root
B. C7 Nerve root
Rationale: The semimembranosus tendon does not C. Axillary nerve
contribute to the pes anserinus. D. Axillary artery
50.What diagnostic test can be used to identify a Correct Answer: C
contracture of the iliotibial band (ITB)?
Rationale: The other structures are more distal than
A. Thomas test the shoulder.
B. Trendelenburg test 55.You are examining a child with the following
C. Ober test deformity of the foot: the foot inversion, forefoot
D. None of the above adduction, and plantar flexion. The correct term for
this deformity is:
Correct Answer: C
A. Talipes equinovarus
Rationale: The Ober test is sued to assess ITB
length. B. Talipes calcaneovalgus
51.While assessing the posture of a patient, you C. Talipes valgus
note that the T2 spinous process in the thoracic D. Talipes calcaneus
region appears to be rotated to the left. Which bony Correct Answer: A
landmark would you be using to make this
Rationale: This is the definition of an individual who
approximation of the vertebral level?
walks on the toes.
A. Inferior angle of the scapula
56.You are about to examine a teenage boy who
B. Superior angle of the scapula has been diagnosed with Erb-Duchenne paralysis.
C. Spine of the scapula Which of the following muscles would you expect to
D. Xiphoid process of the sternum find weak?
Correct Answer: B A. Deltoid, and the internal rotators of the shoulder
Rationale: The superior angle of the scapula is used B. Wrist flexors, and finger flexors
for this approximation. C. Wrist and elbow extensors
52. You are evaluating a patient with a diagnosis of D. Biceps, brachialis, supinator, and brachioradialis
hip ligament sprain. Which of the following hip Correct Answer: D
ligaments attaches to the anterior inferior iliac spine
of the pelvis and the intertrochanteric line of the Rationale: Erb-Duchenne paralysis is another name
for a paralysis that involves C5-6.
femur?
A. Ischiofemoral ligament 57. You are examining a patient who sustained a
hand and wrist injury a few weeks prior. You notice
B. Iliofemoral ligament (Y ligament of Bigelow) evidence of atrophy of the muscles of the thenar
C. Pubofemoral ligament eminence. This would indicate an injury to which
D. Ligamentum teres nerve?
Correct Answer: B A. Median
Rationale: This is the description of the iliofemoral B. Ulnar
ligament attachments. The iliofemoral ligament is C. Radial
considered to be the strongest of the hip ligaments. D. Musculocutaneous
53. A patient arrives at your clinic with a diagnosis Correct Answer: A
of congenital coxa valga deformity. This type of
deformity refers to: Rationale: The median nerve innervates the thenar
eminence.
A. An increase in the angle of inclination between
the neck of the femur and its shaft 58. You are examining a patient who has been
diagnosed with a peripheral nerve injury. The
B. An abnormality of the proximal end of the tibia patient presents with the following findings: an
C. A decrease in the angle of inclination between inability to fully flex the index finger and middle
the neck of the femur and itsshaft finger, loss of thumb opposition, and loss of
D. An abnormality of the distal end of the tibia sensation in the lateral one half of the ring finger,
the middle and index finger, and the thumb. Which
Correct Answer: A nerve do you suspect is involved?
Rationale: Coxa valga describes an increase in the A. Musculocutaneous
angulation from the normal of 130°.
B. Radial
54. You are examining a patient with a diagnosis of
a shoulder dislocation. Which of the following C. Median
structures is often injured in association with a D. Ulnar
shoulder dislocation? Correct Answer: C
A. Radial nerve Rationale: The median nerve innervates the muscles
B. Radial artery that produce finger flexion, thumb opposition, and
sensation to the ring finger.
59. You are applying manual muscle testing to a balance. Compromise to which structure could
patient’s wrist and hand to help determine the cause these complaints:
extent of recovery from a median nerve lesion. A. Internal carotid artery
Which of the following muscles is not supplied by
the median nerve? B. Vertebral artery
C. Ligamentum flavum
A. Flexor carpi radialis
B. Flexor digitorum superficialis D. B and C
Correct Answer: D
C. Flexor pollicus longus
D. Abductor pollicus longus Rationale: These symptoms can be caused by
compromise to the vertebral artery and by buckling
Correct Answer: D of the ligamentum flavum during cervical extension.
Rationale: The abductor pollicus longus is
innervated by the posterior interosseous nerve.
60. You are examining the ligaments of the knee.
Which of the following statements below best
describes the function of the anterior cruciate
ligament?
A. It prevents excessive anterior translation of the
tibia on the femur
B. It prevents valgus (contact) stress at the knee
C. It prevents excessive posterior translation of the
tibia on the femur
D. It prevents varus (contact) stress at the knee
Correct Answer: A
Rationale: The anterior cruciate ligament prevents
excessive anterior translation of the tibia on the
femur.
61. You decide to assess the joint glide involved
with ankle dorsiflexion in a patient. Using your
knowledge of the convex- concave rule, which of
the following joint mobility techniques would be the
most appropriate to perform?
A. Posterior glide of the talus on the mortise of the
distal tibiofibular articulation
B. Anterior glide of the talus on the calcaneus
C. Posterior glide of the talus on the calcaneus
D. Anterior glide of the talus on the mortise of the
distal tibiofibular articulation
Correct Answer: A
Rationale: Ankle dorsiflexion involves a posterior
glide of the convex talus on the concave ankle
mortise.
62. You are assessing ankle plantar flexor strength.
All of the following muscles produce plantar flexion,
except:
A. Fibularis brevis
B. Tibialis posterior
C. Popliteus
D. Soleus
Correct Answer: C
Rationale: The popliteus serves to externally rotate
the femur/internally rotate the tibia and assists with
knee flexion.
63.You are examining the active range of motion of
a patient’s cervical spine. During cervical extension,
the patient complains of dizziness and a loss of

You might also like